Đến nội dung

Hình ảnh

Nếu $x^{\frac{p-1}{2}} \equiv 1 \mathbb{mod p}$ thì $x$ là số chính phương

- - - - -

  • Please log in to reply
Chủ đề này có 6 trả lời

#1
bangbang1412

bangbang1412

    Độc cô cầu bại

  • Phó Quản lý Toán Cao cấp
  • 1667 Bài viết

Chứng minh rằng nếu $x^{\frac{p-1}{2}} \equiv 1 \mathbb{mod p}$ từ một lúc nào đó với các số nguyên tố $p$ ( hay nói cách khác là số chính phương $\mathbb{mod p}$ ) thì $x$ là số chính phương.


Bài viết đã được chỉnh sửa nội dung bởi bangbang1412: 17-06-2018 - 20:27

$$[\Psi_f(\mathbb{1}_{X_{\eta}}) ] = \sum_{\varnothing \neq J} (-1)^{\left|J \right|-1} [\mathrm{M}_{X_{\sigma},c}^{\vee}(\widetilde{D}_J^{\circ} \times_k \mathbf{G}_{m,k}^{\left|J \right|-1})] \in K_0(\mathbf{SH}_{\mathfrak{M},ct}(X_{\sigma})).$$


#2
Tuanmysterious

Tuanmysterious

    Binh nhất

  • Thành viên mới
  • 34 Bài viết
P/S: Ý là anh bảo cm định lý này à

#3
MoMo123

MoMo123

    Sĩ quan

  • Điều hành viên THCS
  • 334 Bài viết

Chứng minh rằng nếu $x^{\frac{p-1}{2}} \equiv 1 \mathbb{mod p}$ từ một lúc nào đó với các số nguyên tố $p$ ( hay nói cách khác là số chính phương $\mathbb{mod p}$ ) thì $x$ là số chính phương 

Đây là lời giải của em: Ta có :$ (x,p)=1$

Xét 2 trường hợp sau:

TH1: $p=2$ , thì x lẻ, Xét số $x=2t+1$ và số $s=2q+1$

Ta có: $s^2-x= (2q+1)^2-(2t+1) =2(2q^2+2q-t) \vdots 2$

Nên $s^2 \equiv x (mod p)$( đpcm)

TH2: $p$ lẻ Ta có:

Với mỗi số $k \in \left \{ 1,2,3,...,p-1 \right \}$ tồn tại duy nhất một số $k' \in \left\{1,2,3,...,p-1 \right \}$ sao cho $kk' \equiv x(mod p)$

Xét 2 trường hợp sau:

$+_{1})$ Nếu $k=k'$ thì $x \equiv k^2$ từ đây suy ra x là số chính phương mod p

$+_{2})$ Nếu với mọi k ta luôn có $ k\neq k'$ , Khi đó tập $\left\{1,2,3,...,p-1 \right \}$ sẽ được chia thành $\frac{p-1}{2}$ tập con $\left \{k;k' \right \}$ sao cho $ kk' \equiv a$

Nên $a^{\frac{p-1}{2}} \equiv (p-1)!$

Theo định lí Wilson , $(p-1)! \equiv -1$

Nên ta có $1 \equiv -1 (mod p)$

Vì p lẻ nên điều này là không thể. Vậy chỉ có $+_{1}$ xảy ra (đpcm)


Bài viết đã được chỉnh sửa nội dung bởi MoMo123: 18-06-2018 - 23:12


#4
bangbang1412

bangbang1412

    Độc cô cầu bại

  • Phó Quản lý Toán Cao cấp
  • 1667 Bài viết

Đây là lời giải của em: Ta có :$ (x,p)=1$

Xét 2 trường hợp sau:

TH1: $p=2$ , thì x lẻ, Xét số $x=2t+1$ và số $s=2q+1$

Ta có: $s^2-x= (2q+1)^2-(2t+1) =2(2q^2+2q-t) \vdots 2$

Nên $s^2 \equiv x (mod p)$( đpcm)

TH2: $p$ lẻ Ta có:

Với mỗi số $k \in \left \{ 1,2,3,...,p-1 \right \}$ tồn tại duy nhất một số $k' \in \left\{1,2,3,...,p-1 \right \}$ sao cho $kk' \equiv x(mod p)$

Xét 2 trường hợp sau:

$+_{1})$ Nếu $k=k'$ thì $x \equiv k^2$ từ đây suy ra x là số chính phương mod p

$+_{2})$ Nếu với mọi k ta luôn có $ k\neq k'$ , Khi đó tập $\left\{1,2,3,...,p-1 \right \}$ sẽ được chia thành $\frac{p-1}{2}$ tập con $\left \{k;k' \right \}$ sao cho $ kk' \equiv a$

Nên $a^{\frac{p-1}{2}} \equiv (p-1)!$

Theo định lí Wilson , $(p-1)! \equiv -1$

Nên ta có $1 \equiv -1 (mod p)$

Vì p lẻ nên điều này là không thể. Vậy chỉ có $+_{1}$ xảy ra (đpcm)

Anh không hiểu em viết gì, đề bài là nếu tồn tại số nguyên $n$ sao cho $x$ là số chính phương mod $p$ với mọi số nguyên tố $p > n$ thì $x$ phải là một số chính phương.


$$[\Psi_f(\mathbb{1}_{X_{\eta}}) ] = \sum_{\varnothing \neq J} (-1)^{\left|J \right|-1} [\mathrm{M}_{X_{\sigma},c}^{\vee}(\widetilde{D}_J^{\circ} \times_k \mathbf{G}_{m,k}^{\left|J \right|-1})] \in K_0(\mathbf{SH}_{\mathfrak{M},ct}(X_{\sigma})).$$


#5
MoMo123

MoMo123

    Sĩ quan

  • Điều hành viên THCS
  • 334 Bài viết

Anh không hiểu em viết gì, đề bài là nếu tồn tại số nguyên $n$ sao cho $x$ là số chính phương mod $p$ với mọi số nguyên tố $p > n$ thì $x$ phải là một số chính phương.

Cái em đang chứng minh ở đây là $x$ là một số chính phương $mod p$ nếu như $x^{\frac{p-1}{2}} \equiv 1 mod p$ :D . Còn cái x là số chính phương  thì là phần ở đây :  https://artofproblem...munity/c6h64322


Bài viết đã được chỉnh sửa nội dung bởi MoMo123: 20-06-2018 - 11:37


#6
Donald Trump

Donald Trump

    Binh nhất

  • Thành viên mới
  • 28 Bài viết

Sử dụng bổ đề sau : Nếu $a$ không phải là số chính phương thì tồn tại vô hạn số nguyên tố $p$ sao cho $a$ không phải là số chính phương mod $p$

Chứng minh tham khảo IMOSL 2009 N7



#7
NHoang1608

NHoang1608

    Sĩ quan

  • Thành viên
  • 375 Bài viết

bài shortlist 2009 N7 không sử bổ đề này mà. Nhầm với bài $a,b$ là số nguyên dương thỏa mãn $ab$ không là số chính phương. Chứng minh rằng tồn tại vô hạn $n$ nguyên dương sao cho $(a^{n}-1)(b^{n}-1)$ không là số chính phương. Bổ đề trên có thể chứng minh bằng thặng dư trung hoa với $drichlet$. Phản chứng ngược lại bổ đề ta có đpcm.


Bài viết đã được chỉnh sửa nội dung bởi NHoang1608: 20-06-2018 - 20:08

The greatest danger for most of us is not that our aim is too high and we miss it, but that it is too low and we reach it.

----- Michelangelo----





1 người đang xem chủ đề

0 thành viên, 1 khách, 0 thành viên ẩn danh